50 Points! Multiple choice algebra graphing question. The related graph of a quadratic equation is shown at the right. Use the graph to determine the solutions of the equation. Photo attached. Thank you!

50 Points! Multiple Choice Algebra Graphing Question. The Related Graph Of A Quadratic Equation Is Shown

Answers

Answer 1

Answer:

(C) {-3, 2} is the correct answer.


Related Questions

NO LINKS!! URGENT HELP PLEASE!!
Select all that apply

b. Symmetric with respect to the x-axis

Answers

The ones that are symmetric with respect to the x-axis is:

y = -7x^2

Checking the symmetric for all equations

A function is symmetric with respect to the x-axis if replacing y with -y in the equation does not change the equation. In other words, if the graph of the function is the same when reflected across the x-axis.

y = -7x^2 is symmetric with respect to the x-axis, since replacing y with -y gives -(-y) = y and the equation remains the same.y = 6x² - 9 is not symmetric with respect to the x-axis, since replacing y with -y gives -(-y) = y, but the equation changes to -y = 6x² - 9, which is not the same as the original equation.x=1/4y^2 is not a function, since it does not pass the vertical line test and has multiple values of x for some values of y.y=x^3-1 is not symmetric with respect to the x-axis, since replacing y with -y gives -(-y) = y, but the equation changes to -y = x^3 - 1, which is not the same as the original equation.x=-y^2+9 is not symmetric with respect to the x-axis, since replacing y with -y gives -(-y) = y, but the equation changes to x = -(-y)^2 + 9, which is not the same as the original equation.y=sqrt(x) is not symmetric with respect to the x-axis, since replacing y with -y gives -(-y) = y, but the equation changes to -y = sqrt(x), which is not the same as the original equation.y=sqrt(x)-6 is not symmetric with respect to the x-axis, since replacing y with -y gives -(-y) = y, but the equation changes to -y = sqrt(x) - 6, which is not the same as the original equation.

Therefore, only the equation y = -7x^2 is symmetric with respect to the x-axis.

Learn more about symmetric here:

https://brainly.com/question/14405062

#SPJ1

The table shows the length of the songs played by a radio station during a 90-minute period. Alicia is making a histogram of the data. What frequency should she show for the interval 160-169 seconds?

Answers

The frequency for the interval 160-169 seconds is 3.

What is mode of the data?

The value that appears the most frequently in a data set is its mode. In the data set, it is the number that appears the most frequently. For instance, in the subsequent data set:

2, 3, 4, 4, 5, 5, 5, 6, 7, 8

Since no other value appears more frequently, the mode is 5, which only appears three times. The data set is considered to have several modes if various values occur with the same greatest frequency. There is no mode if no value appears more than once in the data collection.

From the given table we see that the songs that have the time duration between 160 and 169 is:

162, 168, 165

Hence, the frequency for the interval 160-169 seconds is 3.

Learn more about mode here:

https://brainly.com/question/30891252

#SPJ1

The complete question is:

A quadratic function yields negative values between x = 2 and x = 6. Its minimum value is −2. What are the coordinates of the y-intercept? Enter your answer by filling in the boxes.

Answers

Answer:

Since the quadratic function has a minimum value at some point between x = 2 and x = 6, its graph is a downward-facing parabola.

Let's assume that the function is of form f(x) = ax^2 + bx + c, where a, b, and c are constants.

Since the minimum value of the function is −2, we know that the vertex of the parabola lies on the line y = -2. Also, we know that the x-coordinate of the vertex is the average of 2 and 6, which is 4.

Therefore, the equation of the parabola can be written as f(x) = a(x-4)^2 - 2.

Since the y-intercept is the value of y when x = 0, we can find it by plugging in x = 0 into the equation of the parabola:

f(0) = a(0-4)^2 - 2

f(0) = 16a - 2

We know that the function yields negative values between x = 2 and x = 6, so the parabola must intersect the y-axis below the x-axis. This means that the y-intercept is negative.

To find the y-intercept, we need to solve the equation 16a - 2 = 0, which gives us a = 1/8.

Therefore, the equation of the parabola is f(x) = (1/8)(x-4)^2 - 2.

Finally, we can find the y-intercept by plugging in x = 0:

f(0) = (1/8)(0-4)^2 - 2

f(0) = 8 - 2

f(0) = 6

So the coordinates of the y-intercept are (0, 6).

Find the value of x.

Answers

Applying the intersecting secants theorem, the value of x is calculated as: x = 5.

What is the Intersecting Secants Theorem?

According to the intersecting secants theorem, if two secant lines intersect outside of a circle, then the product of the length of one secant segment and its external segment is equal to the product of the length of the other secant segment and its external segment.

Using the theorem, the equation below is created to find the value of x:

4(4 + 5) = (x - 2)(x - 2 + x + 4)

4(9) = (x - 2)(2x + 2)

36 = 2x² - 2x - 4

2x² - 2x - 4 - 36 = 0

2x² - 2x - 40 = 0

Factorize:

(x + 4)(x - 5) = 0

x = -4 or x = 5

Learn more about the intersecting secants theorem on:

https://brainly.com/question/31033097

#SPJ1

In a spring, the tension (Tnewtons) is directly proportional to its extension ( x cm). When the tension is 150 newtons, the extension is 6 cm. (a) Find a formula for T' in terms of x. T = AD

Answers

The formula for tension (T) in terms of extension (x) is T = 25x.

The given statement is T = kx, where T is the tension, x is the extension, and k is the proportionality constant.

To solve this problem, we need to find the value of k. We can do that by substituting the given values of T and x into the equation:

T = kx

150 = k×6

k = 25

Therefore, the formula for tension (T) in terms of extension (x) is T = 25x.

To learn more about the proportional relationship visit:

brainly.com/question/12917806.

#SPJ1

Use the expression below to complete the following tasks.
(3a2 - 5ab + b2) - (-3a2 + 2b2 + 8ab)
What is the additive inverse of the polynomial being subtracted?After you rewrite subtraction as addition of the additive inverse, how can the like terms be grouped?

Answers

With additive inverse and by grouping like terms the simplified form of the expression is 6a²-13ab-b².

The given expression is 3a²-5ab+b²-(-3a²+2b²+8ab).

Here,

3a²-5ab+b²+3a²-2b²-8ab

Group like terms, that is

(3a²+3a²)+(-5ab-8ab)+(b²-2b²)

= 6a²-13ab-b²

Therefore, with additive inverse and by grouping like terms the simplified form of the expression is 6a²-13ab-b².

Learn more about the addition of algebraic expressions here:

https://brainly.com/question/21443100.

#SPJ1


PLEASE HELP MEE‼️‼️
Tell me which each 3 boxes should I pick for each one of them PLEASE READ them I beg (proportional relationships)

Answers

number one take the second


Help!!!!!!!!!!!!
Write the new coordinates of A'B'C' after
the reflection described below
A(5, -1), B(3,-4), C(3, 0)
Reflection across the y-axis
A:
B:
C:

Answers

Thus, new coordinates of A'B'C' after the reflection across the y-axis are-

A'(-5, -1), B'(-3,-4), C'(-3, 0).

Explain about the reflection across y-axis:

The x-coordinate remains constant when a point is reflected across the x-axis, but the y-coordinate is assumed to be the additive inverse. Point (x, y) is reflected across the x-axis as (x, -y).

The y-coordinate stays the same when a point is reflected across the y-axis, but the x-coordinate is assumed to be the additive inverse. Point (x, y) is reflected across the y-axis as (-x, y).

Given coordinates of ABC.

A(5, -1), B(3,-4), C(3, 0)

Reflection across the y-axis : y coordinates remains same while x coordinate multiplied with -1.

(x,y) -- > (-x, y)

A'(-5, -1), B'(-3,-4), C'(-3, 0)

Thus, new coordinates of A'B'C' after the reflection across the y-axis are-

A'(-5, -1), B'(-3,-4), C'(-3, 0).

Know more about the reflection across y-axis:

https://brainly.com/question/15175017

#SPJ1

A flagpole is 12 feet fall. Its shadow is
11 feet long. How far is it from the top of the flagpole to the end of its shadow?

Answers

The distance from the top of the flagpole to the end of its shadow is approximately 10.02 feet.

Explain the term distance

Distance refers to the measurement of the space between two objects or points. It is typically measured in units such as meters, kilometres, miles, or feet. The distance can be calculated using various methods, including using maps, GPS technology, or mathematical formulas.

According to the given information

We can set up the following proportion:

h / 12 = d / 11

We can cross-multiply to get:

h x 11 = 12 x d

Simplifying further:

d = (h x 11) / 12

We need to solve for d, so we need to find the value of h. Using the Pythagorean theorem, we can set up the following equation:

h² + d² = 12²

Substituting d = (h x 11) / 12, we get:

h² + ((h x 11) / 12)² = 12²

Simplifying:

h² + (121h²) / 144 = 144

Multiplying both sides by 144/265:

265h² / 144 = 144

Solving for h:

h² = (144 x 144) / 265

h = √(20736 / 265)

h = √(78.113)

Now we can substitute this value into our earlier equation to find d:

d = (√(78.113) x 11) / 12

d ≈ 10.02 feet

Therefore, the distance from the top of the flagpole to the end of its shadow is approximately 10.02 feet.

To know more about distance visit

brainly.com/question/15172156

#SPJ1

if k(x) = 3x, then f'(x)=? A. x³Ln3 B. 3xLn3 C. 3x/Lnx D. 3/3xLn3​

Answers

The correct option is B .solution of given problem with the help of integrating the given function is 3xLn3

what is integration and function ?

The area under a curve in a given range can be calculated mathematically via integration. To locate the region between the curve and the x-axis, it is necessary to find a function's antiderivative and evaluate it twice.

A function is a rule that gives each input value a distinct output value. It can be compared to a machine that processes inputs into outputs in accordance with a predetermined rule or formula.

According to given information

To find f'(x), we need to take the derivative of f(x), where f(x) is the antiderivative of k(x).

Since k(x) = 3x, we can find f(x) by integrating 3x with respect to x:

f(x) = ∫ 3x dx = 3/2 x² + C

where C is a constant of integration.

Now we can find f'(x) by taking the derivative of f(x):

f'(x) = d/dx (3/2 x² + C) = 3x

Therefore, the answer is (B) 3xLn3. Option (A) is incorrect because there is no natural logarithm term in the derivative of f(x). Option (C) is incorrect because the derivative of 3x is 3, not 3/Ln(x). Option (D) is incorrect because there is no x in the denominator of the natural logarithm term.

To know more about integration Visit:

brainly.com/question/31433890

#SPJ1

In a survey 7 out of 8 people preferred cooking to washing dishes what percentage of the people surveyed preferred cooking

Answers

The percentage of the people that preferred cooking who where surveyed would be = 87.2%

How to calculate the percentage of people who preferred cooking?

The total number of people surveyed = 8

The total number of people that preferred cooking = 7

The percentage of people that preferred cooking;

= 7/8×100/1

= 700/8

= 87.2%

Learn more about percentage here:

https://brainly.com/question/24339661

Luis's cedar chest measures 4 ft long 2 ft wide and 2 1/4 ft high. What is the volume of the chest? PLEASE USE THE FORMULA IN THE STEP BY STEP EXPLANATION! I REALLY NEED HELP!!!!!!!!!!!!!!!!!!!!!!!!!!!!!!!!!!!!!!!!!!!

Answers

the formula for volume is length x width x height. you would have to multiply all three of those together (i think, i havent had a math class in yearss )

Q3: Use the image to determine the direction and angle of rotation.

graph of triangle ABC in quadrant 4 and a second polygon A prime B prime C prime in quadrant 3

90° clockwise rotation
90° counterclockwise rotation
180° clockwise rotation
360° counterclockwise rotation

Answers

the direction and angle of rotation between the two polygons is 180° clockwise rotation.

How to solve the question?

Based on the given information, we can determine the direction and angle of rotation between the two polygons.

First, let's look at the initial positions of the polygons. The graph of triangle ABC is located in Quadrant 4, which means that it is in the bottom-right portion of the coordinate plane. On the other hand, the second polygon A'B'C' is located in Quadrant 3, which is in the bottom-left portion of the coordinate plane.

To find the direction and angle of rotation between the two polygons, we need to imagine rotating one polygon onto the other. We can see that the two polygons are mirror images of each other across the y-axis. Therefore, we can infer that there is a horizontal line of symmetry between the two polygons.

If we rotate polygon A'B'C' 180 degrees clockwise around the origin, it will overlap perfectly with triangle ABC. This is because a 180-degree rotation is equivalent to a half-turn or a flip, which is exactly what we need to make the two polygons overlap. Therefore, the answer is 180° clockwise rotation.

In summary, the direction and angle of rotation between the two polygons is 180° clockwise rotation.

To know more about polygon visit :-

https://brainly.com/question/26583264

#SPJ1

please help pleaseee i need dis good grade

Answers

Answer:62.8 units

Step-by-step explanation:

Find the length of the third side. If necessary, write in simplest radical form.
3√3 and 6

Answers

Answer: 3

Step-by-step explanation:

You would need to use the Pythagorean theorem to solve this equation. 6 is the hypotenuse and 3[tex]\sqrt{3}[/tex] is the longer leg.  

a^2+b^2=c^2

c= hypotenuse

6^2=3[tex]\sqrt{3}[/tex]^2 + a^2

36=27+a^2

36-27=9

[tex]\sqrt{9}[/tex] = 3

if you multiplied a number by 1/2 , the result would be Responses

Answers

Answer:

half the number you started with

Step-by-step explanation:

8 times 1/2 would be 4....6 times 1/2 would be 3!

It would be like divided a number by 2.

Someone pls help me I’m so confused

Answers

The number of triangles that are displayed can be found to be 216 triangles.

How to find the number of triangles ?

Each rectangular tile has a decorative pattern of 3 equal-sized squares, and each square is divided into 2 same-sized triangles. Therefore, each tile has 3 squares * 2 triangles per square = 6 triangles.

Marnie uses 36 of these tiles on the wall behind her kitchen stove. To find the total number of triangles displayed, multiply the number of triangles per tile by the total number of tiles:

Total triangles = 6 triangles per tile x 36 tiles = 216 triangles

So, there are 216 triangles displayed on the wall behind Marnie's kitchen stove.

Find out more on triangles at https://brainly.com/question/17488511

#SPJ1

100 Points! Algebra question, photo attached. Only looking for an answer to B. Please show as much work as possible. Thank you!

Answers

Answer:f/g=29

Step-by-step explanation:

Answer: (f/g)=29

Hope this helps

Juliet tell her in Israel our volunteer firefighters on Saturday to volunteer fire department hell is annual coin drop fundraiser at street light after one hour Keller had collected $42.50 more than Julia. Israel had collected $15 less than Keller. The three firefighters collected $125.95 in total. How much did each person collect?

Part A: Choose all equations that represents this problem. J= the amount Julia collected, K= the amount Keller collected, I = the amount Israel collected.
Options:
A: 125.95=J+(42.5+J)+(J+15)
B: 125.95=K+42.5+K-15
C: 125.95=I+(1+15)+(I-27.5)
D: 125.95=K+(K-42.5)+(K-15)

Part B: Keller collected 42.50 more than Julia, and Israel had collected 15 less than Keller. The three firefighters collected $125.95 in total. How much did each person collect?
Options:

$125.95
$61.15
$46.15
$42.50
$64.80
$18.65
$15.00

Answers

Part A: A

Part B: Julia collected 18.65, Keller collected 61.15, and Israel collected 45.15

determina el volumen cuyo diametro es de 8 y su altura de 15 cm

Answers

Answer:

3016 centímetros cúbicos.

Step-by-step explanation:

El radio del cilindro es de 8 cm y la altura es de 15 cm. Sustituya 8 por r y 15 por h en la fórmula . Simplifique. Por lo tanto, el volumen del cilindro es de alrededor de 3016 centímetros cúbicos.

Determine how many integer solutions there are to

x₁ + x₂ + x3 + x₁ = 20, if
0≤x₁ < 3, 0≤ x₂ < 4, 0≤x3 <5, 0≤x4 < 6

Answers

Based on the information given, there are a total of 118 solutions.

How many possible solutions are there?

This is a problem of solving a Diophantine equation subject to some conditions. Let's introduce a new variable y4 = 20 - (x1 + x2 + x3 + x4). Then the problem can be restated as finding the number of solutions to:

x1 + x2 + x3 + y4 = 20

Subject to the following conditions:

0 ≤ x1 < 3

0 ≤ x2 < 4

0 ≤ x3 < 5

0 ≤ y4 < 6

We can solve this problem using the technique of generating functions. The generating function for each variable is:

(1 + x + x^2) for x1

(1 + x + x^2 + x^3) for x2

(1 + x + x^2 + x^3 + x^4) for x3

(1 + x + x^2 + x^3 + x^4 + x^5) for y4

The generating function for the equation is the product of the generating functions for each variable:

(1 + x + x^2)^3 (1 + x + x^2 + x^3 + x^4 + x^5)

We need to find the coefficient of x^20 in this generating function. We can use a computer algebra system or a spreadsheet program to expand the product and extract the coefficient. The result is: 1118

Learn more about solutions in https://brainly.com/question/30665317

#SPJ1

Answer: This problem involves finding the number of non-negative integer solutions to the equation x₁ + x₂ + x3 + x₁ = 20 subject to the given constraints. We can use the stars and bars method to solve this problem.

Suppose we have 20 stars representing the sum x₁ + x₂ + x3 + x₁. To separate these stars into four groups corresponding to x₁, x₂, x₃, and x₄, we need to place three bars. For example, if we have 20 stars and 3 bars arranged as follows:

**|**||

then the corresponding values of x₁, x₂, x₃, and x₄ are 2, 4, 6, and 8, respectively. Notice that the position of the bars determines the values of x₁, x₂, x₃, and x₄.

In general, the number of ways to place k identical objects (stars) into n distinct groups (corresponding to x₁, x₂, ..., xₙ-₁) using n-1 separators (bars) is given by the binomial coefficient (k+n-1) choose (n-1), which is denoted by C(k+n-1, n-1).

Thus, the number of non-negative integer solutions to the equation x₁ + x₂ + x3 + x₁ = 20 subject to the given constraints is:

C(20+4-1, 4-1) = C(23, 3) = 1771

However, this count includes solutions that violate the upper bounds on x₁, x₂, x₃, and x₄. To eliminate these solutions, we need to use the principle of inclusion-exclusion.

Let Aᵢ be the set of non-negative integer solutions to the equation x₁ + x₂ + x3 + x₁ = 20 subject to the given constraints, where xᵢ ≥ mᵢ for some integer mᵢ. Then, we want to find the cardinality of the set:

A = A₀ ∩ A₁ ∩ A₂ ∩ A₃

where A₀ is the set of all non-negative integer solutions to the equation x₁ + x₂ + x3 + x₁ = 20, and Aᵢ is the set of solutions that violate the upper bound on xᵢ.

To find the cardinality of A₀, we use the formula above and obtain:

C(20+4-1, 4-1) = 1771

To find the cardinality of Aᵢ, we subtract the number of solutions that violate the upper bound on xᵢ from the total count. For example, to find the cardinality of A₁, we subtract the number of solutions where x₂ ≥ 4 from the total count. To count the number of solutions where x₂ ≥ 4, we fix x₂ = 4 and then count the number of solutions to the equation x₁ + 4 + x₃ + x₄ = 20 subject to the constraints 0 ≤ x₁ < 3, 0 ≤ x₃ < 5, and 0 ≤ x₄ < 6. This count is given by:

C(20-4+3-1, 3-1) = C(18, 2) = 153

Similarly, we can find the cardinalities of A₂ and A₃ by fixing x₃ = 5 and x₄ = 6, respectively. Using the principle of inclusion-exclusion, we obtain:

|A| = |A₀| - |A

Step-by-step explanation:

Where are the x-intercept(s) of the graph?

Answers

The x-intercept of the graph is (0,0).

What is an illustration of an x-intercept on a graph?

On a graph, the x-intercept is the point at which a line crosses the x-axis. At that time, the y coordinate has no value. The y-intercept is the point where the line crosses the y-axis. The x coordinate has no value. For example, y = x + 5 would intersect the x-axis at (-5, 0), forming the x-intercept.

From the figure, it is clear that the line crosses the X-axis at the origin, which means that x - coordinate 0 keeping y -coordinate is also zero.

Which means that the x-intercept of the graph is (0,0).

Learn more about intercept here:

https://brainly.com/question/14180189

#SPJ1

Use the formula KE= mv^2/2 where m=mass, V= velocity, KE = kinetic energy. If dev has a mass of 60kg and is running at a constant velocity with 150 J of KE. What is his velocity?

Answers

Dev's velocity is [tex]\sqrt{5}[/tex]. Thus option B.

What is kinetic energy?

Kinetic energy is a amount of energy possessed when an object is in motion. Such that;

KE = 1/2 m v^2

Where m = mass, v = velocity

It is measured in Joules.

From the given question, we have;

KE = 1/2 m v^2

2KE = m v^2

v^2 = 2KE/ m

      = (2*150)/ 60

      = 300/ 60

      = 5

V = (5)^1/2

The velocity of Dev is B. [tex]\sqrt{5}[/tex].

Learn more about kinetic energy at https://brainly.com/question/25959744

#SPJ1

Andrea is playing a board game with her friends. A player spins the spinner shown below and receives the number of points indicated in the section where the arrow stops. A negative value means a loss of points.
What is the expected payoff, in points, for landing on a space of the board game?

Answers

The expected payoff for landing on a space of the board game is 2.67 points.

How to find the expected payoff?

We need to multiply each possible outcome by its probability of occurring and then add all the products to get the expected payoff.

Let's begin by determining the likelihood of each outcome:

The number 8 appears four times, so the probability of getting an 8 is 4/12 = 1/3.

The number 1 appears four times, so the probability of getting a 1 is also 1/3.

The number -2 appears twice, so the probability of getting a -2 is 2/12 = 1/6.

The number - 4 shows up two times, so the likelihood of getting a - 4 is likewise 1/6.

After that, we add up each outcome by multiplying it by its probability:

Expected payoff = (8 * 1/3) + (8 * 1/3) + (8 * 1/3) + (8 * 1/3) + (1 * 1/3) + (1 * 1/3) + (-2 * 1/6) + (-2 * 1/6) + (-4 * 1/6) + (-4 * 1/6)

Expected payoff = 2.67

As a result, the expected reward for landing on a board game space is 2.67 points.

know more about probability visit :

https://brainly.com/question/11234923

#SPJ1

-7 4/5 divided by X = -5 1/5

Answers

The value of the variable is 3/2

What is a fraction?

A fraction cam simply be described as an expression that is being used to represent the part of a whole number, a whole variable or element.

There are different types of fractions. They are listed as;

Improper fractionsProper fractionsComplex fractionsSimple fractionsMixed fractions

From the information given, we have that;

-7 4/5 divided by X = -5 1/5

convert to improper fractions, we have;

-39/5/x = -26/5

Now, cross multiply the values, we have;

-39/5 = -26x/5

cross multiply

-130x = -195

Divide both sides

x = -195/-130

x = 39/26 = 3/2

Learn about fractions at: https://brainly.com/question/11562149

#SPJ1

On the SAT exam a total of 25 minutes is allotted for students to answer 20 math questions without the use of a calculator. A guidance counselor would like to know if the students in his school are prepared to complete this portion of the exam in the timeframe allotted. To investigate, the counselor selects a random sample of 35 students and administers this portion of the test. The students are instructed to turn in their test as soon as they have completed the questions. The mean amount of time taken by the students is 23.5 minutes with a standard deviation of 4.8 minutes. The counselor would like to know if the data provide convincing evidence that the true mean amount of time needed for all students of this school to complete this portion of the test is less than 25 minutes and therefore tests the hypotheses H0: μ = 25 versus Ha: μ < 25, where μ = the true mean amount of time needed by students at this school to complete this portion of the exam. The conditions for inference are met. What are the appropriate test statistic and P-value?

Answers

The P-value is between 0.025 and 0.05. and t = -1.85

On the SAT exam a total of 25 minutes is allotted for students to answer 20 math questions without the use of a calculator.

Therefore tests the hypotheses:

[tex]H_0[/tex] : μ = 25 versus Ha: μ < 25,

where μ = the true mean amount of time needed by students at this school to complete this portion of the exam.

The alternative hypothesis is:

[tex]H_1:\mu < 25[/tex]

The test statistic is given by:

[tex]t=\frac{x-\mu}{\frac{s}{\sqrt{n} } }[/tex]

The parameters are:

'x' is the sample mean. [tex]\mu[/tex] is the value tested at the null hypothesis.s is the standard deviation of the sample.n is the sample size.

the values of the parameters are:

x = 23.5 , [tex]\mu=25[/tex] , s = 4.8, n = 35

Plug all the values in above formula of t- statistic is:

[tex]t = \frac{23.5-25}{\frac{4.8}{\sqrt{35} } }[/tex]

t = -1.85

Using a t-distribution , with a left-tailed test, as we are testing if the mean is less than a value and 35 - 1 = 34 df, the p-value is of 0.0365.

t = –1.85; the P-value is between 0.025 and 0.05.

Learn more about t-distribution at:

https://brainly.com/question/13574945

#SPJ1

4 divide by 3/5 as a fration

Answers

Answer:

6 and 2/3

Step-by-step explanation:

4 divided by 3/5 is the same as 4 divided by 0.6

4 divided by 0.6 equals 6.6 repeating...

or

6 and 2/3

ben and david plot the locations of their houses and the park on the grid show below. they r planning to each start at home and travel in a straight line to the park. how many times greater is the distance ben travels to get to the park than the distance david travels?

Answers

The distance Ben travels to get to the park is approximately 2.21 times greater than the distance David travels. This is found by using the distance formula to calculate the distances each person travels and then comparing the results.

To solve this problem, we need to find the distance between each person's house and the park using the distance formula

distance = √((x₂ - x₁)² + (y₂ - y₁)²)

For Ben

distance = √((-2 - 4)² + (-2 - 6)²) = √((-6)² + (-8)²) = √(100) = 10

For David

distance = √((-2 - (-7))² + (-2 - (-6))²) = √((5)² + (4)²) = √(41)

So the ratio of Ben's distance to David's distance is

10 / √(41) ≈ 2.21

Therefore, Ben's distance is about 2.21 times greater than David's distance.

To know more about distance:

https://brainly.com/question/30308375

#SPJ1

Springfield will be opening a new high school in the fall. The number of underclassmen (9th and 10th graders) must fall between 600 and 700
(inclusive), the number of upperclassmen (11th and 12th graders) must fall between 500 and 600 (inclusive), and the number of students cannot
exceed 1200. Let a represent the number of underclassmen and let b represent the number of upperclassmen. Write a set of inequalities that
models the constraints on the composition of the student body.
number of underclassmen:
number of upperclassmen:
Total number of students:
:: 600 < a < 700
000
:: 600 ≤ a ≤ 700
:: 500 ≤ b ≤ 600
:: a + b ≤ 1200
:: 500 < b < 600
:: a + b > 1200
= a + b < 1200
:: a + b > 1200

Answers

The correct set of inequalities that model the constraints on the composition of the student body are:

600 ≤ a ≤ 700, 500 ≤ b ≤ 600 and a + b ≤ 1200

What is inequalities?

In mathematics, an inequality is a mathematical statement that indicates that two expressions are not equal. It is a statement that compares two values, usually using one of the following symbols: "<" (less than), ">" (greater than), "≤" (less than or equal to), or "≥" (greater than or equal to).

The correct set of inequalities that model the constraints on the composition of the student body are:

600 ≤ a ≤ 700 (the number of underclassmen must fall between 600 and 700, inclusive)

500 ≤ b ≤ 600 (the number of upperclassmen must fall between 500 and 600, inclusive)

a + b ≤ 1200 (the total number of students cannot exceed 1200)

Note that the inequalities 600 < a < 700 and 500 < b < 600 are not correct, as they do not take into account the inclusive limits of the ranges for the number of underclassmen and upperclassmen. Also, the inequality a + b > 1200 is not correct, as it contradicts the previous inequality a + b ≤ 1200.

To learn more about inequalities from the given link:

https://brainly.com/question/30231190

#SPJ1

A 40" screen television at a popular electronics retailer is priced at $600. The wall mount for this sized television costs $29.99.

Part A: If Jamie purchases the television and the wall mount and has a coupon for 30% off, how much will her subtotal be? Show all necessary work. (4 points)

Part B: If Jamie makes the purchase in a state with a 7% state sales tax, what will her final total be? Show all necessary work. (2 points)

Part C: The electronics retailer is extending a special offer to install the wall mount and television for free. However, Jamie decides to tip the installation specialist 10% of the original purchase price before the discount is applied. How much would her new total be, including tax, discount, and tip? Show all necessary work. (4 points)

Answers

Jamie's subtotal will be $440.99Jamie's total will be $471.86Her new total  including tax, discount, and tip will be $531.86Cost estimation

The cost of the television and wall mount before discount is $600 + $29.99 = $629.99

After a 30% discount, the subtotal is:

$629.99 x 0.70 = $440.99

The sales tax is 7% of the subtotal:

$440.99 x 0.07 = $30.87

The final total is the subtotal plus the sales tax:

$440.99 + $30.87 = $471.86

The original purchase price before discount is $600.

10% of $600 is $60.

So Jamie decides to tip the installation specialist $60.

After the discount, the subtotal is $440.99 (as calculated in Part A).

The sales tax is 7% of the subtotal:

$440.99 x 0.07 = $30.87

The new total is the subtotal plus the sales tax and the tip:

$440.99 + $30.87 + $60 = $531.86

More on cost estimation can be found here: https://brainly.com/question/14347037

#SPJ1

Other Questions
__________ __________ vehicles tend to have positive offset rims to allow for proper clearance of the hub assembly within the vehicle wheel well. The old and comfortable way is to shave a little here and add a little there. Well, that's not acceptable anymore. I think this great and historic Congress knows that way is no longer acceptable. President Ronald Reagan April 28, 1981 Which adjective best describes President Reagans word choice in this excerpt? a. technicalb. academicc. informal d. indecisive the staple crop for japanese agriculture from the earliest period was a. wet rice. b. hemp. c. wheat. d. millet. e. tea. Question 13 points Successful oligopolit firms must: O Maximize profits in the short run Establish the price leadership role Undercut cartelpricing rules Predict rivali's responses to price and quanti How did the lake stickleback fish lose its spikes? A genealogical firm called Q-L produces its output using the following production function: f(x1,x2)=x11+2x2f(x1,x2)=x11+2x2 where x1x1 denotes the amount of factor 1 and x2x2 denotes the amount of factor 2. The market price of factor 1 is $2 and the market price of factor 2 is $10.(a) To produce y=5y=5 units of output with the lowest cost, how much factor 1 and factor 2 should the firm use? Find this optimal bundle of inputs and calculate the minimal cost. The article is the hunger games, i need 4 sentences please, some of the elements given are: survival, sacrifice, power, poverty, entertainment, corruption.The assignment is this below According to the article what are some of the story elements and themes that create a story that has abiding interest Think about your favorite books movies and TV shows Write a short response that describes how a modern day book movie or TV show that you enjoy reflects at leastPlease help, i need this tonight give you brainless!!! Sam is completing his college program in 3 years and receiveshis degree. This program is usually done in 4 years. Under what circumstances would receiving his college degree in two years have a negativeeffect on his employability and/or salary? I NEED HELP ASAP!!!How can the strengths and weaknesses of each side (the union and the confederacy) contribute to theoutcome of the war? The coordinates of P are (-3, 8) and the coordinates of Q are (9, -2). Find the equation of the line parallel to PQ that passes through the point (6, -1). A choking adult becomes unresponsive while you are doing abdominal thrusts for severe choking. What should you do next? I really need a multi body paragraph on the changes on Greg Ridlys behavior over the course of the story of The Treasurer of Lemon Brown. If you can write it that would be such a big help and would be really appreciated Goals that bring optimal happiness have been seen to be all the following except: Nucleic Acid is tested with DISCHE REAGENT which contains 1g ________dissolved in 2.5 mL concentrated H2SO4 and the volume is brought up to 100mL with _________ _______ _______(both of these reagents are colorless) Proponents of the idea of the "living" Constitution would likely support the concept of A down counter output instruction wi decrement by 1 each time the counted event occurs. true/false People value the ability to connect with a driver rapidly, rather than waiting for a scarce taxicab. And people (with a nice enough car) can earn extra cash giving rides, working as little or as much as they please. Today, an estimated 2 million people drive for Uber, often evenings and weekends after theyre finished the day at a full-time job. Explain how the growth of Uber reflects the way in which ridesharing has altered the opportunity cost of leisure. Think of a routine task that you did without question. How can the task be improved and benefit you? Use the drop-down menus to answer each question.During which part of the water cycle does water from Earth's surface move into the atmosphere?What happens as water cools? |How does water return to Earth from the atmosphere? [IntroDone 1. Consider the Heckscher-Ohlin model with two factors of production, labour L and capital K, and two goods, cars c and shoes s. Denote the factor prices by r for capital and w for labour. The production function for cars is given by 1-ac Fr(Kc, Lc) = KL?- and for shoes by F (K,, L3) = KL-as. Suppose that 1 > ac > as > 0. = (a) Derive the optimal factor input ratios Kg/L, for computers and shoes (g = c, s), depending on the relative factor price r/w. Which good is labour intensive? Can there be a factor intensity reversal? (b) Draw a graph showing the possible relationship between the factor price ratio . and optimal factor input ratios Kg/L, for computers and shoes (g =c, s). Add the curve for the relative aggregate factor demand (Kc+K3)/(Lc + Ls) to your graph (it is a weighted average of the two goods' relative factor demands) and mark the equilibrium factor price (w*/r*) in your graph. A country Home, with factor endowments KH and LH, opens up to free international trade of goods. Suppose that K/L" < KW/LW, where KW and LW are the factor endowments in the world. (c) Which good will Home export? Compared to the autarky equilibrium: will (i) the relative price of shoes, (ii) capital intensity in car production, (iii) capital intensity in shoe production and (iv) the relative factor price r/w have increased or decreased? Explain. (d) Compared to autarky, has the real wage (in terms of shoes and computers) increased or decreased? Explain. (e) Real world trade shows that economies both import and export both cars and shoes. Is this observation consistent with the model? Propose a suitable modi- fication of the model to account for this empirical fact. 1. Consider the Heckscher-Ohlin model with two factors of production, labour L and capital K, and two goods, cars c and shoes s. Denote the factor prices by r for capital and w for labour. The production function for cars is given by 1-ac Fr(Kc, Lc) = KL?- and for shoes by F (K,, L3) = KL-as. Suppose that 1 > ac > as > 0. = (a) Derive the optimal factor input ratios Kg/L, for computers and shoes (g = c, s), depending on the relative factor price r/w. Which good is labour intensive? Can there be a factor intensity reversal? (b) Draw a graph showing the possible relationship between the factor price ratio . and optimal factor input ratios Kg/L, for computers and shoes (g =c, s). Add the curve for the relative aggregate factor demand (Kc+K3)/(Lc + Ls) to your graph (it is a weighted average of the two goods' relative factor demands) and mark the equilibrium factor price (w*/r*) in your graph. A country Home, with factor endowments KH and LH, opens up to free international trade of goods. Suppose that K/L" < KW/LW, where KW and LW are the factor endowments in the world. (c) Which good will Home export? Compared to the autarky equilibrium: will (i) the relative price of shoes, (ii) capital intensity in car production, (iii) capital intensity in shoe production and (iv) the relative factor price r/w have increased or decreased? Explain. (d) Compared to autarky, has the real wage (in terms of shoes and computers) increased or decreased? Explain. (e) Real world trade shows that economies both import and export both cars and shoes. Is this observation consistent with the model? Propose a suitable modi- fication of the model to account for this empirical fact.